How Much Paint for a Hemispherical Dome Using Differentials?

Click For Summary
SUMMARY

The discussion focuses on estimating the amount of paint required for a hemispherical dome with a diameter of 54 meters, using differentials. The surface area of the hemisphere is calculated using the formula 2πr², where r is the radius. The differential equation dy = 4πr dx is applied, resulting in a paint volume estimate of approximately 16.96 cubic meters when using a thickness of 0.05 cm. It is emphasized that all measurements must be in consistent units for accurate calculations.

PREREQUISITES
  • Understanding of differential calculus
  • Familiarity with surface area formulas, specifically for hemispheres
  • Knowledge of unit conversions, particularly between centimeters and meters
  • Basic algebra skills for manipulating equations
NEXT STEPS
  • Study the application of differentials in real-world scenarios
  • Learn more about surface area calculations for various geometric shapes
  • Explore unit conversion techniques in mathematical contexts
  • Investigate practical applications of calculus in estimating material requirements
USEFUL FOR

Students in calculus courses, mathematics educators, and professionals involved in construction or painting projects requiring precise material estimations.

maladroit
Messages
42
Reaction score
0

Homework Statement



Use differentials to estimate the amount of paint needed to apply a coat of paint 0.05 cm thick to a hemispherical dome with diameter 54 m.

Homework Equations



dy=dy/dx *dx

Surface area of a hemishpere=2pi*r^2

The Attempt at a Solution



dy=4pi*r dx
dy=4pi*27 *.05
dy=16.96
 
Physics news on Phys.org
You want to express all of your numbers in the same units. 0.05 cm is not the same as 0.05 m.
 
genius...thank you!
 
Question: A clock's minute hand has length 4 and its hour hand has length 3. What is the distance between the tips at the moment when it is increasing most rapidly?(Putnam Exam Question) Answer: Making assumption that both the hands moves at constant angular velocities, the answer is ## \sqrt{7} .## But don't you think this assumption is somewhat doubtful and wrong?

Similar threads

  • · Replies 9 ·
Replies
9
Views
2K
  • · Replies 5 ·
Replies
5
Views
3K
  • · Replies 2 ·
Replies
2
Views
4K
  • · Replies 1 ·
Replies
1
Views
2K
  • · Replies 6 ·
Replies
6
Views
12K
  • · Replies 5 ·
Replies
5
Views
11K
  • · Replies 7 ·
Replies
7
Views
8K
  • · Replies 4 ·
Replies
4
Views
31K
Replies
4
Views
11K
  • · Replies 3 ·
Replies
3
Views
3K